You are on page 1of 14

Decision sciences 2

End term answer key

Question 1: (15 points)

Perumal Nadar & Sons (PNS) is a manufacturer of Ayurvedic supplements prepared as per the
ancient books on Ayurveda. One of their product is Chyawanprash prepared using sugar, ghee,
honey, amla (Indian Gooseberry), and herbs. Few additional ingredients are added based on
variants of Chyawanprash. PNS is interested in finding the optimal product mix for two variants
of Chyawanprash: (a) Regular and (b) Fruit and Nut. Ingredients such as sugar, honey and ghee
are readily available in the market; however, supply is constrained for ingredients such as
ashwagandha, amla, and Tabasheer. The quantity of ingredients required for different types of
Chyawanprash is shown in Table 1 per 1 kg of Chyawanprash. The profit earned from regular and
fruit and nut flavour per kg is 100 and 140 respectively.
Table 1 Data related to different Chyawanprash and requirements of ingredients per 1 Kg of different
Chyawanprash

Chyawanprash Availability

Regular Fruit and Nut

Ashwagandha (in grams) 80 120 12,000

Amla (in grams) 160 140 20,000

Tabasheer (in grams) 120 180 24,000

(a) Formulate an LP model to maximize the profit for PNS. Clearly define all the decision variables,
objective function and constraints. (2 points)

Decision variables

X1 = amount of regular chyawanprash

X2 = amount of fruit and net chyawanprash

Objective function Max 100 X1 + 140 X2


Subject to constraints

80X1 + 120 X2 <= 12000

160X1 + 140X2 <= 20000

120X1 + 180X2 <= 24000

X1, X2 >= 0

(b) Use the graphical method to find the optimal solution and optimal objective function value.
(6 points)

From graph, points ABCO forms feasible region. Corner points of the feasible region are

A (0, 100) B (90, 40) C (125, 0) O (0, 0).

Substituting these points in objective function,

A (0, 100) 14000


B (90, 40) 14600

C (125, 0) 12500

O (0, 0) 0

Optimal solution X1 = 90 X2 = 40

Optimal objective function value 14600

(c) What is the worth of one gram of Tabasheer? Explain. (1 point)

Tabasheer is a non-binding constraint because the shadow price is zero. Also, as the constraint is outside

the feasible region and there is a slack. (120 X1 + 180 X2 = 120*90 + 180 * 40 = 18000; X1 = 90, X2 = 40).

Therefore, worth of one gram of Tabasheer is zero.

(d) What is the maximum increase in profit for Regular Chyawanprash for which the current optimal
solution remains optimal? (2 points)

Slope of the binding constraints are -80/120 and -160/140.

Slope of the objective function –X1/140.

-80/120 >= -X1/140 >= -160/140

80/120 <= X1/140 <= 160/140

2/3 <= X1/140 <= 8/7

93.33 <= X1 <= 160

Maximum increase in profit for regular chyawanprash for which the current optimal solution remains

Optimal is 160 -100 = 60.


(e) Find the worth of one gram of Amla. What is the range of availability of Amla for which this worth
per gram will be valid? Show all work. (4 points)

Constraints,

80X1 + 120 X2 = 12000 ………… (1)

160X1 + 140X2 = 20000 ………….. (2)

Increasing R.H.S by 1,

160X1 + 140X2 = 20001……….. (2) (Increasing R.H.S by 1)

160X1 + 240X2 = 24000………… (1) *2

Solving we get, X1 = 90.015, X2 = 39.99 and the corresponding objective function value is 14600.1

Shadow price = Change in objective function/change in R.H.S = 0.1/1 = 0.1

(14600.1-14600 = 0.1)

Thus, worth of one gram of amla is 0.1.

Another way of solving is, we can use dual problem method.

Dual : min z = 12000Y1 + 20000Y2 + 24000Y3

Subject to constraints,

80Y1 + 160Y2 + 120 Y3 >=100

120Y1 + 140Y2 +180Y3 >=140

We know that worth of Tabasheer is zero, thus Y3 = 0.

80Y1 + 160Y2 =100

120Y1 + 140Y2 =140

Solving we get, Y1 = 1.05 and Y2 = 0.1

Thus worth of one gram of amla is 0.1.

Range of availability of amla is given by intersection of amla and ashwagandha constraint (150, 0),

(0, 100).

160X1 + 140X2 = k1 at (150,0) k1= 24000

160X1 + 140X2 = k2 at (0,100) k2=14000

Thus allowable increase is = 24000-20000 = 4000


allowable decrease is = 14000-20000 = 6000

Therefore, range = (14000,24000)

Question 2: (20 points)

Ajanta Garments is a fast-growing export company. The cutting and stitching sections can each handle
100,000 man-hours per week, and the finishing section 50,000 man-hours. The packaging section can
handle 250,000 pieces per week. They produce two types of shirts, formal and casual; two types of
trousers, also formal and casual, as well as kurtas. The contribution from these are Rs.130, 80, 150, 110
and 125 per piece, respectively. The time required in each section is given in the table below. One of their
valued customers has an order of 5,000 sets where each set consists of 2 formal shirts and 1 formal
trouser. The net contribution per set is Rs.400.
Table: Time in hours
Formal shirt Casual Shirt Formal Trouser Casual Trouser Kurta
Cutting 1 0.5 1.5 1 1.5
Stitching 2 1.5 2 1.5 1
Finishing 0.5 0.5 0.75 0.75 0.25

Ajanta Garments wants to decide the product mix and formulates the problem as follows:

Max z = 130x1+80x2+150x3+110x4+125x5+400w
Subject to:
x1+0.5x2+1.5x3+x4+1.5x5+3.5w <= 100,000 (cutting)
2x1+1.5x2+2x3+1.5x4+1x5+6w <= 100,000 (Stitching)
0.5x1+0.5x2+0.75x3+0.75x4+0.25x5+1.75w <= 50,000 (Finishing)
w >= 5000 (Demand for set)
All variables >= 0.
The Solver output for the formulation is given below:

Variable Cells
Final Reduced Objective Allowable Allowable
Cell Name Value Cost Coefficient Increase Decrease
$B$7 var x1 11250 0 130 120 2.857
$C$7 var x2 -2.5 80 2.5 1E+30
$D$7 var x3 -10 150 10 1E+30
$E$7 var x4 -2.5 110 2.5 1E+30
$F$7 var x5 47500 0 125 10 10
$G$7 var w 400 20 1E+30

Constraints
Final Shadow Constraint Allowable Allowable
Cell Name Value Price R.H. Side Increase Decrease
$H$3 Cutting 100000 100000 22500 47500
$H$4 Stitching 100000 100000 95000 15000
$H$5 Finishing 26250 50000 1E+30 23750
$H$6 Set 5000 5000 4090.91 5000

(a) In the given formulation, explain the coefficient of w in the constraint for cutting.(1 point)

Ans. The coefficients is simply the time taken to cut two formal shirts and one formal
trouser. For cutting, formal shirts take 1 hour and trousers take 1.5 hours. Since each set
has two shirts and one trouser, the total time taken is 3.5

(b) Find the optimal value of the decision variable w. (1 point)

Ans. Since the constraint is w>= 5000 and is tight, the value of w is 5000

(c) Find the values of decision variables x2, x3 and x4 in the optimal solution. Explain clearly.
(1 point)
Ans. Since their reduced costs are non zero, these variables are all zero.

(d) What is the optimal objective function value? (1 point)

Ans. The objective function value will be –


11250x130 +47500x125 +5000x400 = 94,00,000

(e) Find the shadow prices of all the constraints. (6 points)

Ans. Since constraint for finishing is not binding, we get dual variable y3 = 0.
Since x1, x5 and w are all non zero, we get the corresponding dual constraints as
binding:
y1 + 2y2 = 130
1.5y1 + y2 = 125
3.5y1 + 6y2 – y4 = 400
Solving the first two we get y1 = 60 and y2 =35.
From the third we get y4 = -20. (As the constraint is –W ≤ −5000. )
So, the shadow prices are - y1 = 60 , y2 =35 , y3 = 0 , y4 = - 20 .

(f) Ajanta Garments can outsource the cutting activity at INR 50 per hour. Is it worth outsourcing the
cutting for additional 20000 hours? What will be impact of this on the objective function value?
Explain clearly. (2 points)
Ans. Here the shadow price is INR 60 while the cost of outsourcing the cutting will
be INR 50. So per hour of cutting the profit will increase by (60-50) = INR 10.
Also the allowable increase for cutting is 22500 which is greater than 20000. So,
they can outsource the cutting.
The objective function will increase by – 20000 × 10 = INR 200000.
The new value of the objective function will be – 9400000 + 200000 = 9600000.

(g) If they increase the contribution of x1 andw by 30 and 5 respectively,and decrease the
contribution of x5 by 5, will the optimal solution change?Why or why not? (2 points)
Ans. For simultaneous change we use the 100% rule –
30/120 + 5/20 + 5/10 = 1/4 + 1/4 + 1/2 = 1
We know if change ≤ 1 then the optimal does not change.
So, here also the optimal solution will remain unchanged.

(h) The CEO is trying to decide whether to drop the order of 5000 sets received from the valued
customer. In the long run, they are confident that they can get other customers. What would
you advise him? Give reasons. (2 points)

Ans. As the allowable decrease is 5000 and the shadow price is – 20 they gain 5000 × 20
Net gain would be 100,000. They can either renegotiate the price, which shows an
allowable increase of 20, or if they are confident that there will be no adverse
impact, they can drop the customer.

(i) Many business analysts are suggesting that the company should move up the value chain and
make premium shirts. The contribution for premium shirt would be 175 per shirt. Since the
company does not have skilled tailors, each piece of premium shirt would take 1.5 hours in the
cutting section, 3 hours in the stitching section and the same time in the finishing section. Would
you advise them to go in for the premium shirts? (2 points)

Ans. The reduced cost is 175 – 1.5 × y1 – 3 × y2 = 175 – 1.5 × 60 – 3 ×35


= 175 – 195 = - 20 .
Based on this, we would not advise them to make the premium shirts.

(j) The CEO feels that it will take 6 months to acquire the skills provided they decide to start making
the premium shirts. Once that is done, the time taken to make the shirts would be the same as
that for the formal shirts. What would you advise him? (2 points)

The reduced cost would now be 175 – 1 × y1 – 2 × y2 = 175 – 1 × 60 – 2 ×35


= 175 – 130 = 45. At that time they should introduce it.
But meanwhile, they will take a lower profit for 6 months. They need to assess
this and take a long term view. They can also think of increasing capacity.

Question 3: (20 points)

IP Formulation Problem

A company has one warehouse and supplies retail stores in several locations around the city. The
demand is on 5 stretches of roads and totals 113 tons each day. They decide to buy their own mini
trucks, each of 5 ton capacity. The company has just one product.

(i) Formulate a mixed integer program to minimize the number of trucks

Let w = number of trucks


X = quantity shipped
Min w
St.
X >= 113
x-5w <= 0
x>= 0, w integer
(ii) What is the number of trucks they should buy?

W = 113/5 rounded up or w = 23

After a few days they realize that there are delays in delivering the products. There are a lot of one way
streets as well. After some analysis they find that the demand can be categorized into 5 stretches of the
city. These stretches form a ring with the warehouse in the middle. They label the stretches as 1-2, 2-3,
3-4, 4-5, and 5-1. Each truck can service at most two stretches a day and return by the end of the day to
the warehouse. Given the one way streets and the number of stretches a truck can do in a day, they
decide that the following 10 routes are possible, where node s refers to the start node:

s-1-2- s, s-1-2-3-s, s-2-3-s, s-2-3-4-s, s-3-4-s, s-3-4-5-s, s-4-5-s, s-4-5-1-s, s-5-1-s, s-5-1-2-s.

Thus each truck can deliver products on at most 2 contiguous stretches. For instance a truck that leaves
from the warehouse and goes to node 1, can deliver products to stretch 1-2, and 2-3 at most in a day.
For ease of notation they number the routes from 1 to 10 in the order given above. The demand on each
stretch is given below.

Stretch 1-2 2-3 3-4 4-5 5-1


Demand in tons 25 18 17 21 32

A new intern formulates the problem and gets the solution as 23 trucks with shipments as given below.

Original s-1-2- s-1-2- s-2- s-2-3- s-3- s-3-4- s-4-5- s-4-5- s-5-1- s-5-1-
route s 3-s 3-s 4-s 4-s 5-s s 1-s s 2-s
Route 1 2 3 4 5 6 7 8 9 10
Tons 25 18 0 17 0 0 21 0 32 0
shipped

The total shipments are met and equal 113 and there are 23 trucks which is 113/5 rounded up. He is
very happy that in spite of the restrictions on routes, the number of stretches they can deliver on a
truck per day, and the one way streets, the number of trucks has not gone up. However, someone
looks closer at the solution and finds that 23 is not sufficient. He discovers that the number of
trucks from the solution for each of the routes with non-zero shipment is at least 5, 3.6, 3.4, 4.2, 6.4
which add up to 22.6 – which rounded up gives 23 trucks. But in reality, Route 2 would need 4 trucks
instead of 3.6. When he added up all the trucks needed it came to 5+4+4+5+7 = 25. He was
wondering if he could do better.

(iii) Formulate a mixed integer program for the company. Hint: you may need more variables.

Let xi = quantity in tons shipped on route i, where i = 1, …, 10


Let yi = number of trucks on route i, where i = 1,… 10
Let w = total number of trucks
Min w
s.t.
x1+x10 = 25; x2+x3 = 18; x4+x5 = 17, x6+x7 = 21, x8+x9 = 32
(Note that stretch 1-2 can be served by Routes 1 and 10, stretch 2-3 b y routes 2 and 3 and so on)
xi – 5yi <= 0 this ensures the right number of trucks on each route
SUM(yi over i) – w = 0
Xi >= 0, yi, w integer
Note: we can also drop the variable w and use Min SUM(yi) in the objective function.
The intern is relieved when he finds that they can do with 24 trucks instead of 25 and the solution is

Route 1 2 3 4 5 6 7 8 9 10
Tons 25 18 0 17 0 0 21 32 0 0
shipped

Here he noticed that routes 7 and 8 are contiguous and represent s-4-5-s and s-4-5-1-s and total
tons shipped is 21+32 = 53 and can be done by 11 trucks. He is a bit relieved and shows it to the
management. They say that some customers on route 8 are always served in the second half of the
day (route s-4-5-1-s). Given the one way streets they cannot ask the trucks to go in the reverse
direction s-1-5-4-s. After some brainstorming they decide that the tons shipped using the route s-4-
5-1-s should be restricted to 16.

(iv) What additional constraint(s) is needed?

X8 <= 16

Someone then asks whether it is sufficient to minimize the number of trucks or whether they should
minimize the total cost including the cost of transportation. The following table gives the relevant
daily transport costs for each possible route.

Route 1 2 3 4 5 6 7 8 9 10
Cost (Rs) 200 350 300 650 400 700 350 675 500 900

The cost of each truck on the road is Rs.10 lakhs.

(v) Formulate the problem to minimize the total costs. Assume they want to recover the costs
of the new trucks purchased in one year and that they operate 300 days a year.

Define c1 = 200x300/100,000 (in lakhs of rupees) from Table above. Similarly, we can find all other ci’s.

Min SUM(ci.xi) + 10SUM(yi)

s.t.

All other constraints remain the same.

(vi) During the monsoon they find that on one particular day the road s-1 cannot be used. What
additional constraint(s) are needed?

y1+y2+y8+y9 = 0

(vii) They find that some trucks use only one stretch in their daily travel. Such trucks can be used
for other purposes in the afternoon. If they want at least half the trucks to be free like this,
how would the formulation change?

Routes 2,4,6,8,10 are the ones where the trucks do a double leg. For instance if route 2 is used, this
is s-1-2-3-s and goes over stretch 1-2 and 2-3. We need to restrict this.

y1+y3+y5+y7+y9 – 0.5*SUM(yi) >= 0

Or, 0.5(y2+y4+y6+y8+y10) – 0.5(y1+y3+y5+y7+y9) <= 0


Or, y2+y4+y6+y8+y10 –y1+y3+y5+y7+y9 <= 0
(viii) New zoning rules on traffic restrict the daily number of mini trucks on any given stretch to 8.
What additional constraint(s) are needed?

Stretch 1-2: y1+y2+y10 <= 8


Stretch 2-3: y2+y3+y4 <= 8
Stretch 3-4: y4+y5+y6 <= 8
Stretch 4-5: y6+y7+y8 <= 8
Stretch 5-1: y8+y9+y10 <= 8

Question 4: (10 points)

Consider the following integer programming (IP) problem:


Max 5x1 + 5x2
s.t. 3x1 + 2x2 <= 5
2x1 + 3x2 <= 7
x1, x2 integers
a) State the LP relaxation for the IP given above. Include the upper bounds on the variable values.
(1 point)
Max 5x1 + 5x2
s.t. 3x1 + 2x2 <= 5
2x1 + 3x2 <= 7
x1 , x2 >= 0
Upper bound on x1 = 1
Upper bound on x2 = 2
(Or)
0 <= x1<= 1
0 <= x2 <= 2

b) Solve the given IP using the branch and bound (b-a-b) method (use the LR relaxation solution
provided in Appendix). Show all nodes and branches and proceed as follows: (9 points)

- In case both variables need branching, pick the one with the smaller value first.
- Number the nodes in the order they are created.
- Clearly state the solution and objective function value at each node, and whether the value
obtained is a lower or upper bound for the BIP.
- Clearly state when you fathom a node and give the reason for doing so.
- State the optimal solution and objective function value obtained.
LP0
Max 5x1 + 5x2
s.t. 3x1 + 2x2 <= 5
2x1 + 3x2 <= 7

x1=0.2, x2=2.2, Z=21(UB) X1>=1

X1<=0

LP1

Max 5x1 + 5x2 LP2

s.t. 3x1 + 2x2 <= 5 Max 5x1 + 5x2

2x1 + 3x2 <= 7 s.t. 3x1 + 2x2 <= 5

X1=0, x2=2.33, Z=11.67(UB) 2x1 + 3x2 <= 7

X2<=2 X1=1, x2=1, Z=10(LB)

X2>=3 Fathomed, since integer


solution found.
LP3 LP4

Max 5x1 + 5x2 Max 5x1 + 5x2

s.t. 3x1 + 2x2 <= 5 s.t. 3x1 + 2x2 <= 5

2x1 + 3x2 <= 7 2x1 + 3x2 <= 7

X1=0, x2=2, Z=10(LB) Z=10, Fathomed since


infeasible
There are 2 optimal solutions x1=0, x2=2 & x1=1, x2=1 and objective function value = 10

Question 5: (10 points)

A large part of rural India is dependent on “hand pumps” for their water needs. These work well for water
depths up to 30 meters and do not need much maintenance. Electric/diesel pumps that work beyond this
depth are not viable in remote areas since electricity and diesel might not be available always.

Smart Pumps Co. has come up with solar pumps as a solution. For these, a high yield (at least 2000
litres/hour capacity) potable water bore well is fitted with a small solar water pumping system and an
elevated tank. The storage tank provides piped water to the doorstep of the households. The system does
away with the need for electricity, diesel and batteries, and potentially has a long life.
The company has won a contract with the Ministry of New and Renewable Energy (MNRE) for installation
of solar pumps in various states for both irrigation and drinking water purposes. The deliverables at the
end of Jan, Feb, March and April in 2018 are 2000, 6000, 6000, and 6000 respectively. December’s ending
inventory is expected to be zero.

The company has a production capacity of 5000, 4000, 8000 and 5000 pumps in Jan, Feb, March and April
2018. Management would like a production plan that meets the following goals:

1. There are no monthly ending inventories. This is desirable since the inventory carrying costs are
high.

2. Fluctuations in monthly production levels during these months are avoided. This is important
since changing production levels attracts additional costs.

a) Formulate the production planning problem as a goal programming problem, assuming that having
no monthly ending inventories goal is of higher priority. Define all variables clearly and state the
objective function. Include goal and any other constraints. (8 points)

Variable definition: Xi – production in i th month

i = 1,2,3,4

Inventory variables

+ - +
I1 = d1 - d1 I1 = d1 I1 = 0

+ - +
I2 = d2 - d2 or I2 = d2 or I2 = 0

+ - +
I3 = d3 - d3 I3 = d3 I3 = 0

+ - +
I4 = d4 - d4 I4 = d4 I4 = 0

- - - -
Where d1 = d2 = d3 = d4 = 0

X1<=5000, X2<= 4000, X3<= 8000, X4<= 5000.

Non negativity constraint X1, X2, X3, X4 >= 0 and I1, I2, I3, I4 >= 0.

X1 = 2000 + I1, X2 + I1 = 6000 + I2, X3+I2 = 6000+I3, X4+I3 = 6000+I4


+ + + +
Minimize d1 + d2 + d3 + d4

+ -
X2 – X1 = d5 - d5

+ -
X3 – X2 = d6 - d6

+ -
X4– X3 = d7 - d7

Ii = achievement value or P1 >> P2

+ + + +
Minimize d1 + d2 + d3 + d4

b) Now assume that both the goals are equally important. How would the formulation above
change? (2 points)

+ + + + + + + +
Minimize d1 + d2 + d3 + d4 + d1 + d2 + d3 + d4

You might also like